(16 intermediate revisions by 2 users not shown)
Line 1: Line 1:
 +
[[Category:ECE]]
 +
[[Category:QE]]
 +
[[Category:CNSIP]]
 +
[[Category:problem solving]]
 +
[[Category:automatic control]]
 +
[[Category:optimization]]
 +
 
= [[ECE PhD Qualifying Exams|ECE Ph.D. Qualifying Exam]] in "Automatic Control" (AC)  =
 
= [[ECE PhD Qualifying Exams|ECE Ph.D. Qualifying Exam]] in "Automatic Control" (AC)  =
  
= Question 3, Part 2, August 2011 =
+
= [[ECE-QE_AC3-2011|Question 3, August 2011]], Part 5=
  
 
:[[ECE-QE AC3-2011 solusion-1|Part 1]],[[ECE-QE AC3-2011 solusion-2|2]],[[ECE-QE AC3-2011 solusion-3|3]],[[ECE-QE AC3-2011 solusion-4|4]],[[ECE-QE_AC3-2011_solusion-5|5]]
 
:[[ECE-QE AC3-2011 solusion-1|Part 1]],[[ECE-QE AC3-2011 solusion-2|2]],[[ECE-QE AC3-2011 solusion-3|3]],[[ECE-QE AC3-2011 solusion-4|4]],[[ECE-QE_AC3-2011_solusion-5|5]]
Line 33: Line 40:
 
The '''KKT condition (FONC) '''for local minimizer&nbsp;<span class="texhtml">''x''<sup> * </sup></span>&nbsp;of <span class="texhtml">''f''</span>&nbsp;is:  
 
The '''KKT condition (FONC) '''for local minimizer&nbsp;<span class="texhtml">''x''<sup> * </sup></span>&nbsp;of <span class="texhtml">''f''</span>&nbsp;is:  
  
<font color="#ff0000"><span style="font-size: 17px;">'''<math>\text{1. } \mu^{*}\geq0</math>'''</span></font>  
+
<font color="#ff0000"><span style="font-size: 17px;">'''&nbsp; &nbsp; &nbsp;&nbsp;<math>\text{1. } \mu^{*}\geq0</math>'''</span></font>  
  
<font color="#ff0000"><span style="font-size: 20px;">'''<math>\text{2. } Df\left ( x^{*} \right )+\lambda ^{*T}Dh\left ( x^{*} \right )+\mu ^{*T}Dg\left ( x^{*} \right )=0^{T}</math>'''</span></font>  
+
<font color="#ff0000"><span style="font-size: 20px;">'''&nbsp; &nbsp; &nbsp;<math>\text{2. } Df\left ( x^{*} \right )+\lambda ^{*T}Dh\left ( x^{*} \right )+\mu ^{*T}Dg\left ( x^{*} \right )=0^{T}</math>'''</span></font>  
  
<font color="#ff0000"><math>\text{3. } \mu ^{*T}g\left ( x^{*} \right )=0</math><br></font>  
+
<font color="#ff0000">&nbsp; &nbsp; &nbsp; &nbsp;&nbsp;<math>\text{3. } \mu ^{*T}g\left ( x^{*} \right )=0</math><br></font>  
  
<font color="#ff0000"></font><math>\text{4. } h\left ( x^{*} \right )=0</math>  
+
<font color="#ff0000"></font>&nbsp; &nbsp; &nbsp; &nbsp;&nbsp;<math>\text{4. } h\left ( x^{*} \right )=0</math>  
  
<math>\text{5. } g \left( x^{*}  \right) \leq0</math><br>  
+
&nbsp; &nbsp; &nbsp; &nbsp;&nbsp;<math>\text{5. } g \left( x^{*}  \right) \leq0</math><br>  
  
'''SONC: '''Suppose that&nbsp;<span class="texhtml">''x''<sup> * </sup></span>&nbsp;is regular<br> <math>\text{1. } \mu ^{*}\geq0 \text{, } Df\left ( x^{*} \right )+\lambda ^{*T}Dh\left ( x^{*} \right )+\mu ^{*T}Dg\left ( x^{*} \right )=0^{T} \text{, } \mu ^{*T}g\left ( x^{*} \right )=0</math>
+
'''Definision: Regular point''' &nbsp; &nbsp; &nbsp;&nbsp; &nbsp; &nbsp; &nbsp;&nbsp;
  
<math>\text{2. For all } y\in T\left( x^{*} \right ) \text{, we have } y^{T}L\left ( x^{\ast },\mu ^{\ast }, \lambda ^{\ast }\right )y\geq 0</math>  
+
&nbsp; &nbsp; &nbsp; &nbsp; &nbsp;<math>x^{*} \text{ satisfy } h\left( x^{*} \right)=0, g\left( x^{*} \right)\leq0 \text{ and let } J\left(x^{*}\right)= \left \{  j:g_{j}\left(x^{*}\right)=0 \right \}</math>  
  
<math>T\left( x^{* } \right)= \left \{ y\in\Re^{n}: Dh\left( x^{*} \right)y=0, Dg_{j}\left( x^{*} \right)y=0, j\in \tilde{J}\left( x^{*} \right)  \right \}</math>  
+
'''&nbsp; &nbsp; &nbsp; &nbsp; &nbsp;<math>x^{*}\text{ is regular point if } \nabla h_{i} \left( x^{*} \right),  \nabla g_{j} \left( x^{*} \right),  1\leq i\leq m, j\in J \left( x^{*} \right)</math> '''
 +
 
 +
'''SONC: '''Suppose that&nbsp;<span class="texhtml">''x''<sup> * </sup></span>&nbsp;is regular<br> &nbsp; &nbsp; &nbsp; &nbsp;&nbsp;<math>\text{1. } \mu ^{*}\geq0 \text{, } Df\left ( x^{*} \right )+\lambda ^{*T}Dh\left ( x^{*} \right )+\mu ^{*T}Dg\left ( x^{*} \right )=0^{T} \text{, } \mu ^{*T}g\left ( x^{*} \right )=0</math>
 +
 
 +
&nbsp; &nbsp; &nbsp; &nbsp;&nbsp;<math>\text{2. For all } y\in T\left( x^{*} \right ) \text{, we have } y^{T}L\left ( x^{\ast },\mu ^{\ast }, \lambda ^{\ast }\right )y\geq 0</math>
 +
 
 +
&nbsp; &nbsp; &nbsp; &nbsp; &nbsp; &nbsp; &nbsp; &nbsp;<math>T\left( x^{* } \right)= \left \{ y\in\Re^{n}: Dh\left( x^{*} \right)y=0, Dg_{j}\left( x^{*} \right)y=0, j\in J\left( x^{*} \right)  \right \}</math>
 +
 
 +
&nbsp; &nbsp; &nbsp; &nbsp; &nbsp; &nbsp; &nbsp; &nbsp;<math>J\left(x^{*}\right)= \left \{   j:g_{j}\left(x^{*}\right)=0 \right \}</math>  
  
 
'''SOSC:''' There exist a feasible point&nbsp;<span class="texhtml">''x''<sup> *&nbsp;<sub></sub><sub></sub></sup></span>that&nbsp;  
 
'''SOSC:''' There exist a feasible point&nbsp;<span class="texhtml">''x''<sup> *&nbsp;<sub></sub><sub></sub></sup></span>that&nbsp;  
  
<math>\text{1. } \mu ^{*}\geq0 \text{, } Df\left ( x^{*} \right )+\lambda ^{*T}Dh\left ( x^{*} \right )+\mu ^{*T}Dg\left ( x^{*} \right )=0^{T} \text{, } \mu ^{*T}g\left ( x^{*} \right )=0</math>  
+
&nbsp; &nbsp; &nbsp; &nbsp;&nbsp;<math>\text{1. } \mu ^{*}\geq0 \text{, } Df\left ( x^{*} \right )+\lambda ^{*T}Dh\left ( x^{*} \right )+\mu ^{*T}Dg\left ( x^{*} \right )=0^{T} \text{, } \mu ^{*T}g\left ( x^{*} \right )=0</math>
 +
 
 +
&nbsp; &nbsp; &nbsp; &nbsp;&nbsp;<math>\text{2. For all } y\in \tilde{T}\left( x^{* }\mu^{*} \right) \text{, we have } y^{T}L\left ( x^{\ast },\mu ^{\ast }, \lambda ^{\ast }\right )y\geq 0</math>  
  
<math>\text{2. For all } y\in \tilde{T}\left( x^{* }\mu^{*} \right) \text{, we have } y^{T}L\left ( x^{\ast },\mu ^{\ast }, \lambda ^{\ast }\right )y\geq 0</math>  
+
&nbsp; &nbsp; &nbsp; &nbsp; &nbsp; &nbsp; &nbsp; &nbsp;<math>\tilde{T}\left( x^{* },\mu^{*} \right)= \left \{ y: Dh\left( x^{*} \right)y=0, Dg_{i}\left( x^{*} \right)y=0, i\in \tilde{J}\left( x^{*},\mu^{*} \right)   \right \}</math>  
  
<math>\tilde{T}\left( x^{* },\mu^{*} \right)= \left \{ y: Dh\left( x^{*} \right)y=0, Dg_{i}\left( x^{*} \right)y=0, i\in \tilde{J}\left( x^{*},\mu^{*} \right)  \right \}</math>  
+
&nbsp; &nbsp; &nbsp; &nbsp; &nbsp; &nbsp; &nbsp;&nbsp;<math>\tilde{J}\left ( x^{\ast },\mu ^{\ast } \right )= \left \{ i:g_{i}\left ( x^{\ast } \right ) = 0,\mu_{i}^{\ast }> 0\right \}</math>  
  
 
'''Process:'''  
 
'''Process:'''  
  
a. Write down the KKT condition for this probelm  
+
&nbsp; &nbsp; &nbsp; a. Write down the KKT condition for this probelm  
  
b. Find all points (and KKT multipliers) satisfying the KKT condition. In each case, determine if the point is regular.  
+
&nbsp; &nbsp; &nbsp; b. Find all points (and KKT multipliers) satisfying the KKT condition. In each case, determine if the point is regular.  
  
c. Find all points in part b that also satisfy the SONC.  
+
&nbsp; &nbsp; &nbsp; c. Find all points in part b that also satisfy the SONC.  
  
d. Find all points in part c that also satisfy the SOSC.  
+
&nbsp; &nbsp; &nbsp; d. Find all points in part c that also satisfy the SOSC.  
  
e. Find all points in part c that are local minimizers.  
+
&nbsp; &nbsp; &nbsp; e. Find all points in part c that are local minimizers.  
  
 
----
 
----
Line 111: Line 128:
 
\end{bmatrix} \text{, }</math><br>  
 
\end{bmatrix} \text{, }</math><br>  
  
<math>\left\{\begin{matrix}
+
&nbsp; &nbsp; &nbsp; &nbsp; &nbsp; &nbsp;&nbsp;<math>\left\{\begin{matrix}
 
\nabla l\left( x,\mu \right)=\begin{pmatrix}
 
\nabla l\left( x,\mu \right)=\begin{pmatrix}
 
-4+\mu_{2}-\mu_{3}\\  
 
-4+\mu_{2}-\mu_{3}\\  
Line 194: Line 211:
  
 
<math>\color{red} y\in \tilde{T}\left( x^{* }\mu^{*} \right) \text{, where } \tilde{T}\left( x^{* },\mu^{*} \right)= \left \{ y:Dg_{i}\left( x^{*} \right)y=0, i\in \tilde{J}\left( x^{*},\mu^{*} \right)  \right \}</math>  
 
<math>\color{red} y\in \tilde{T}\left( x^{* }\mu^{*} \right) \text{, where } \tilde{T}\left( x^{* },\mu^{*} \right)= \left \{ y:Dg_{i}\left( x^{*} \right)y=0, i\in \tilde{J}\left( x^{*},\mu^{*} \right)  \right \}</math>  
 +
 +
<math>\color{red} \tilde{J}\left ( x^{\ast },\mu ^{\ast } \right ) \text{ has constrain for } \mu ^{\ast } \text{, }  \tilde{J}\left ( x^{\ast },\mu ^{\ast } \right )= \left \{ i:g_{i}\left ( x^{\ast } \right ) = 0,\mu_{i}^{\ast }> 0\right \}</math>
 +
 +
<math>\color{red} \tilde{J}\left ( x^{\ast },\mu ^{\ast } \right ) \subset 
 +
J\left(x^{*}\right)</math>, &nbsp; &nbsp;<math>\color{red} T\left( x^{* } \right) \subset \tilde{T}\left( x^{* },\mu^{*} \right)</math>
  
 
----
 
----
Line 199: Line 221:
 
<math>\color{blue}\text{Solution 2:}</math>  
 
<math>\color{blue}\text{Solution 2:}</math>  
  
<font color="#ff0000"><span style="font-size: 17px;">'''<math>L\left ( x_{1}\mu \right )= D^{2} l \left ( x _{1}\mu \right )= \begin{bmatrix} 2+2\mu_{1} & 0 \\ 0 & 2 \end{bmatrix}</math>'''</span></font>  
+
<font color="#ff0000"><span style="font-size: 17px;">'''<math>L\left ( x_{1},\mu \right )= D^{2} l \left ( x _{1},\mu \right )= \begin{bmatrix} 2+2\mu_{1} & 0 \\ 0 & 2 \end{bmatrix}</math>'''</span></font>  
  
 
&nbsp; &nbsp; &nbsp; &nbsp; &nbsp; &nbsp; &nbsp; &nbsp; &nbsp; &nbsp;<math>\text{for point } x^{*}=\begin{bmatrix} 0 \\ 0 \end{bmatrix} \text{, we get } \mu_{1}=-2  \text{ from KKT condition.}</math>  
 
&nbsp; &nbsp; &nbsp; &nbsp; &nbsp; &nbsp; &nbsp; &nbsp; &nbsp; &nbsp;<math>\text{for point } x^{*}=\begin{bmatrix} 0 \\ 0 \end{bmatrix} \text{, we get } \mu_{1}=-2  \text{ from KKT condition.}</math>  
  
<font color="#ff0000" size="5">'''<math>\therefore L \left ( x^{*}, \mu ^{*}\right )=\begin{bmatrix} -2 & 0 \\ 0 & 2 \end{bmatrix}</math><br>'''</font>  
+
<font color="#ff0000" size="5">'''&nbsp; &nbsp; &nbsp; &nbsp; &nbsp; &nbsp;<math>\therefore L \left ( x^{*}, \mu ^{*}\right )=\begin{bmatrix} -2 & 0 \\ 0 & 2 \end{bmatrix}</math><br>'''</font>  
  
 
<font color="#ff0000"><span style="font-size: 20px;">'''<math>\tilde{T}\left( x^{* },\mu^{*} \right)= \left \{ y:Dg_{i}\left( x^{*} \right)y=0, i\in \tilde{J}\left( x^{*},\mu^{*} \right)  \right \}</math>'''</span></font>  
 
<font color="#ff0000"><span style="font-size: 20px;">'''<math>\tilde{T}\left( x^{* },\mu^{*} \right)= \left \{ y:Dg_{i}\left( x^{*} \right)y=0, i\in \tilde{J}\left( x^{*},\mu^{*} \right)  \right \}</math>'''</span></font>  
  
<math>\tilde{J}\left ( x^{\ast },\mu ^{\ast } \right )= \left \{ i:g_{i}\left ( x^{\ast } \right ) = 0,\mu_{i}^{\ast }> 0\right \}</math>&nbsp; &nbsp; &nbsp;<math>\therefore i= 2</math>  
+
&nbsp; &nbsp; &nbsp; &nbsp; &nbsp; &nbsp; &nbsp;<math>\tilde{J}\left ( x^{\ast },\mu ^{\ast } \right )= \left \{ i:g_{i}\left ( x^{\ast } \right ) = 0,\mu_{i}^{\ast }> 0\right \}</math>&nbsp; &nbsp; &nbsp;<math>\therefore i= 2</math>  
  
<math>\therefore \tilde{T}\left ( x^{\ast },\mu ^{\ast } \right )= \left \{ y:\left [ 1,1 \right ]y= 0 \right \}= \left \{ y:y_{1}= -y_{2} \right \}</math>  
+
&nbsp; &nbsp; &nbsp; &nbsp; &nbsp; &nbsp; &nbsp;<math>\therefore \tilde{T}\left ( x^{\ast },\mu ^{\ast } \right )= \left \{ y:\left [ 1,1 \right ]y= 0 \right \}= \left \{ y:y_{1}= -y_{2} \right \}</math>  
  
<math>\begin{bmatrix}
+
<math>y^{T}L\left ( x^{\ast },\mu ^{\ast } \right )y=\begin{bmatrix}
 
y_{1}&  
 
y_{1}&  
 
y_{2}
 
y_{2}
Line 224: Line 246:
 
&nbsp; &nbsp; &nbsp; &nbsp; &nbsp; &nbsp; &nbsp; &nbsp; &nbsp; &nbsp;&nbsp;<math>-2y_{1}^{2}+2y_{2}^{2}\geqslant 0\cdots \left ( 1 \right )</math>  
 
&nbsp; &nbsp; &nbsp; &nbsp; &nbsp; &nbsp; &nbsp; &nbsp; &nbsp; &nbsp;&nbsp;<math>-2y_{1}^{2}+2y_{2}^{2}\geqslant 0\cdots \left ( 1 \right )</math>  
  
<span class="texhtml">&nbsp; &nbsp; &nbsp; &nbsp; &nbsp; &nbsp; &nbsp; &nbsp; &nbsp; &nbsp; for ''y''<sub>1</sub> = ''y''<sub>2</sub>, &nbsp;(1) is always satisfied.</span>  
+
<span class="texhtml">&nbsp; &nbsp; &nbsp; &nbsp; &nbsp; &nbsp; &nbsp; &nbsp; &nbsp; &nbsp; for ''y''<sub>1</sub> = -&nbsp;''y''<sub>2</sub>, &nbsp;(1) is always satisfied.</span>  
  
 
<math>\therefore \text{For all } y\in \tilde{T}\left( x^{* },\mu^{*} \right) \text{, we have } y^{T}L\left ( x^{\ast },\mu ^{\ast } \right )y\geq 0</math>  
 
<math>\therefore \text{For all } y\in \tilde{T}\left( x^{* },\mu^{*} \right) \text{, we have } y^{T}L\left ( x^{\ast },\mu ^{\ast } \right )y\geq 0</math>  
Line 231: Line 253:
  
 
----
 
----
 
+
----
<math>\color{blue}\text{Relative Problem: }</math>  
+
<math>\color{blue}\text{Related Problem: }</math>  
  
 
<span class="texhtml">''&nbsp; &nbsp; &nbsp; &nbsp; &nbsp; &nbsp; &nbsp; &nbsp; &nbsp; minimize'''&nbsp; &nbsp;'''''<b>&nbsp;− ''x''<sub>2</sub> + (''x''<sub>1</sub> − 1)<sup>2</sup> − 2</b></span>  
 
<span class="texhtml">''&nbsp; &nbsp; &nbsp; &nbsp; &nbsp; &nbsp; &nbsp; &nbsp; &nbsp; minimize'''&nbsp; &nbsp;'''''<b>&nbsp;− ''x''<sub>2</sub> + (''x''<sub>1</sub> − 1)<sup>2</sup> − 2</b></span>  
Line 252: Line 274:
 
<math>\text{From 2, } \mu=1 \text{, and } 2\left ( x_{1}-1 \right )=-1</math><br>  
 
<math>\text{From 2, } \mu=1 \text{, and } 2\left ( x_{1}-1 \right )=-1</math><br>  
  
<font face="serif">''<span class="texhtml">From 3, </span>''x''<sub>1</sub> + ''x''<sub>2</sub> = 2<br>''</font>  
+
<math>\text{From 3, } \left( x_{1}+x_{2} \right) =2</math><font face="serif">''<br>''</font>  
  
 
<math>\text{From above two equations, we obtain a candidate point for the minimizer } x^{*}=\begin{bmatrix}
 
<math>\text{From above two equations, we obtain a candidate point for the minimizer } x^{*}=\begin{bmatrix}
Line 263: Line 285:
 
<math>L \left ( x^{*}, \mu ^{*}\right )= F\left ( x^{*}\right )+ \mu ^{*} G\left ( x^{*}\right )=\begin{bmatrix} 2 & 0 \\ 0 & 0 \end{bmatrix}</math>  
 
<math>L \left ( x^{*}, \mu ^{*}\right )= F\left ( x^{*}\right )+ \mu ^{*} G\left ( x^{*}\right )=\begin{bmatrix} 2 & 0 \\ 0 & 0 \end{bmatrix}</math>  
  
<span class="texhtml">Because μ<sup> * </sup> &gt; 0''&nbsp;'',''w''''e&nbsp;''''h''''a''''v''''e'''</span>'''<br> '''  
+
<span class="texhtml">Because μ<sup> * </sup> &gt; 0''&nbsp;'',''w'''e&nbsp;'''h'''a'''v'''e'''''</span>'''''<br> ''' ''  
  
 
<math>\tilde{T}\left( x^{* },\mu^{*} \right)= \left \{ y: \begin{bmatrix}
 
<math>\tilde{T}\left( x^{* },\mu^{*} \right)= \left \{ y: \begin{bmatrix}
Line 299: Line 321:
 
----
 
----
  
[[ECE PhD Qualifying Exams|Back to ECE Qualifying Exams (QE) page]]
+
[[ECE PhD Qualifying Exams|Back to ECE Qualifying Exams (QE) page]]
 
+
[[Category:ECE]] [[Category:QE]] [[Category:Automatic_Control]] [[Category:Problem_solving]]
+

Latest revision as of 10:11, 13 September 2013


ECE Ph.D. Qualifying Exam in "Automatic Control" (AC)

Question 3, August 2011, Part 5

Part 1,2,3,4,5

 $ \color{blue}\text{5. } \left( \text{20 pts} \right) \text{ Consider the following optimization problem, } $

                            $ \text{optimize} \left(x_{1}-2\right)^{2}+\left(x_{2}-1\right)^{2} $

                        $ \text{subject to } x_{2}- x_{1}^{2}\geq0 $

                                                 $ 2-x_{1}-x_{2}\geq0 $

                                                 $ x_{1}\geq0. $

$ \color{blue} \text{The point } x^{*}=\begin{bmatrix} 0 & 0 \end{bmatrix}^{T} \text{ satisfies the KKT conditions.} $


Theorem:

For the problem:    minimize  $ f \left( x \right) $

                             subject to   $ h \left( x \right) =0 $

                                                $ g \left( x \right) \leq 0 $

The KKT condition (FONC) for local minimizer x *  of f is:

      $ \text{1. } \mu^{*}\geq0 $

     $ \text{2. } Df\left ( x^{*} \right )+\lambda ^{*T}Dh\left ( x^{*} \right )+\mu ^{*T}Dg\left ( x^{*} \right )=0^{T} $

        $ \text{3. } \mu ^{*T}g\left ( x^{*} \right )=0 $

        $ \text{4. } h\left ( x^{*} \right )=0 $

        $ \text{5. } g \left( x^{*} \right) \leq0 $

Definision: Regular point              

         $ x^{*} \text{ satisfy } h\left( x^{*} \right)=0, g\left( x^{*} \right)\leq0 \text{ and let } J\left(x^{*}\right)= \left \{ j:g_{j}\left(x^{*}\right)=0 \right \} $

         $ x^{*}\text{ is regular point if } \nabla h_{i} \left( x^{*} \right), \nabla g_{j} \left( x^{*} \right), 1\leq i\leq m, j\in J \left( x^{*} \right) $

SONC: Suppose that x *  is regular
        $ \text{1. } \mu ^{*}\geq0 \text{, } Df\left ( x^{*} \right )+\lambda ^{*T}Dh\left ( x^{*} \right )+\mu ^{*T}Dg\left ( x^{*} \right )=0^{T} \text{, } \mu ^{*T}g\left ( x^{*} \right )=0 $

        $ \text{2. For all } y\in T\left( x^{*} \right ) \text{, we have } y^{T}L\left ( x^{\ast },\mu ^{\ast }, \lambda ^{\ast }\right )y\geq 0 $

               $ T\left( x^{* } \right)= \left \{ y\in\Re^{n}: Dh\left( x^{*} \right)y=0, Dg_{j}\left( x^{*} \right)y=0, j\in J\left( x^{*} \right) \right \} $

               $ J\left(x^{*}\right)= \left \{ j:g_{j}\left(x^{*}\right)=0 \right \} $

SOSC: There exist a feasible point xthat 

        $ \text{1. } \mu ^{*}\geq0 \text{, } Df\left ( x^{*} \right )+\lambda ^{*T}Dh\left ( x^{*} \right )+\mu ^{*T}Dg\left ( x^{*} \right )=0^{T} \text{, } \mu ^{*T}g\left ( x^{*} \right )=0 $

        $ \text{2. For all } y\in \tilde{T}\left( x^{* }\mu^{*} \right) \text{, we have } y^{T}L\left ( x^{\ast },\mu ^{\ast }, \lambda ^{\ast }\right )y\geq 0 $

               $ \tilde{T}\left( x^{* },\mu^{*} \right)= \left \{ y: Dh\left( x^{*} \right)y=0, Dg_{i}\left( x^{*} \right)y=0, i\in \tilde{J}\left( x^{*},\mu^{*} \right) \right \} $

              $ \tilde{J}\left ( x^{\ast },\mu ^{\ast } \right )= \left \{ i:g_{i}\left ( x^{\ast } \right ) = 0,\mu_{i}^{\ast }> 0\right \} $

Process:

      a. Write down the KKT condition for this probelm

      b. Find all points (and KKT multipliers) satisfying the KKT condition. In each case, determine if the point is regular.

      c. Find all points in part b that also satisfy the SONC.

      d. Find all points in part c that also satisfy the SOSC.

      e. Find all points in part c that are local minimizers.


$ \color{blue}\left( \text{i} \right) \text{Does } x^{*} \text{ satisfy the FONC for minimum or maximum? Where are the KKT multipliers?} $

$ \color{blue}\text{Solution 1:} $

        $ f\left( x \right) = \left(x_{1}-2\right)^{2}+\left(x_{2}-1\right)^{2} $

        $ g_{1}\left( x \right)=x_{1}^{2}-x_{2} $

        $ g_{2}\left( x \right)= x_{1}+x_{2}-2 $

        $ g_{3}\left( x \right)= -x_{1} $

 $ \text{ The problem is to optimize f(x), subject to } g_{1}\leq 0, g_{2}\leq 0, g_{3}\leq 0 $

$ \text{Let } l\left( \mu ,\lambda \right)=\nabla f\left(x \right)+\mu_{1} \nabla g_{1}\left( x \right)+\mu_{2} \nabla g_{2}\left( x \right)+\mu_{3} \nabla g_{3}\left( x \right) $

                      $ =\begin{pmatrix} 2x_{1}-4\\ 2x_{2}-2 \end{pmatrix} +\mu_{1} \begin{pmatrix} 2x_{1}\\ -1 \end{pmatrix}+\mu_{2}+\begin{pmatrix} 1\\ 1 \end{pmatrix}+\mu_{3}+\begin{pmatrix} -1\\ 0 \end{pmatrix} =0 $

$ \mu_{1} g_{1}\left( x \right)+\mu_{2} g_{2}\left( x \right)+\mu_{3} g_{3}\left( x \right) $

            $ = \mu_{1} \left( x_{1}^2-x_{2} \right)+\mu_{2} \left( x_{1}+x_{2}-2 \right)+\mu_{3} \left( -x_{1} \right) =0 $

$ \text{Let } x^{*}=\begin{bmatrix} 0\\ 0 \end{bmatrix} \text{, } $

            $ \left\{\begin{matrix} \nabla l\left( x,\mu \right)=\begin{pmatrix} -4+\mu_{2}-\mu_{3}\\ -2-\mu_{1}-\mu_{2} \end{pmatrix}= \begin{pmatrix} 0 \\ 0\end{pmatrix} \\ -2\mu_{2}=0 \end{matrix}\right. \Rightarrow \left\{\begin{matrix} \mu_{1}=-2\\ \mu_{2}=0\\ \mu_{3}=-4 \end{matrix}\right. $

$ \text{As } \mu^{*}\leq 0, x^{*}\begin{bmatrix} 0\\0 \end{bmatrix} \text{satisfies the FONC for maximum.} $


$ \color{blue}\text{Solution 2:} $

$ \text{ Standard form: optimize} \left(x_{1}-2\right)^{2}+\left(x_{2}-1\right)^{2} $

                                  $ \text{subject to } g_{1}\left( x \right)= x_{1}^{2}-x_{2}\leq0 $

                                                           $ g_{2}\left( x \right)= x_{1}+x_{2}-2\leq0 $

                                                           $ g_{3}\left( x \right)= -x_{1}\leq0 $

$ \text{KKT condition: (1) } Dl\left( \mu ,\lambda \right)=Df\left(x \right)+\mu_{1}Dg_{1}\left( x \right)+\mu_{2}Dg_{2}\left( x \right)+\mu_{3}Dg_{3}\left( x \right) $

                                             $ =\left [ 2x_{1}-4+2\mu_{1}x_{1}+\mu_{2}-\mu_{3}, 2x_{2}-2-\mu_{1}+\mu_{2} \right ]=0 $                                            $ \left ( 2 \right ) \mu^{T}g\left ( x \right )=0 \Rightarrow \mu_{1}\left ( x_{1}^2-x_{2} \right )+\mu_{2}\left ( x_{1}+x_{2}-2 \right ) - \mu_{3}x_{1}=0 $

                                 $ \left ( 3 \right ) \mu_{1},\mu_{2},\mu_{3}\geq 0 \text{ for minimizer} $

                                        $ \mu_{1},\mu_{2},\mu_{3}\leq 0 \text{ for maximizer} $

                                        $ \text{where } \mu^{*}=\begin{bmatrix} \mu_{1}\\ \mu_{2}\\ \mu_{3} \end{bmatrix} \text{ are the KKT multiplier.} $

$ \text{For } x^{*}=\begin{bmatrix} 0\\ 0 \end{bmatrix} \text{, } $       $ \left\{\begin{matrix} \nabla l\left( x,\mu \right)=\begin{pmatrix} -4+\mu_{2}-\mu_{3}\\ -2-\mu_{1}+\mu_{2} \end{pmatrix}=\begin{pmatrix} 0\\0 \end{pmatrix}\\ -2\mu_{2}=0 \end{matrix}\right. \Rightarrow \left\{\begin{matrix} \mu_{1}=-2\\ \mu_{2}=0\\ \mu_{3}=-4 \end{matrix}\right. $

$ \therefore x^{*}=\begin{bmatrix} 0 \\ 0 \end{bmatrix} \text{ satisfy FONC for maximum} $


$ \color{blue}\left( \text{ii} \right) \text{Does } x^{*} \text{ satisfy SOSC? Carefully justify your answer.} $

$ \color{blue}\text{Solution 1:} $

$ L\left ( x^{*},\mu^{*} \right )= \nabla l \left( x^{*},\mu^{*} \right)= \begin{pmatrix} 2&0 \\ 0&2 \end{pmatrix}-2\begin{pmatrix} 2&0 \\ 0&0 \end{pmatrix} = \begin{pmatrix} -2&0 \\ 0&2 \end{pmatrix} $

$ \tilde{T}\left( x^{* }\mu^{*} \right) : \left\{ \begin{matrix} y^{T}\binom{0}{-1} =0 \\ y^{T}\binom{-1}{0} =0 \end{matrix} \right. \Rightarrow \tilde{T}\left( x^{* }\mu^{*} \right)= \left \{ \binom{0}{0} \right \} $

SOSC is trivially satisfied.

$ \color{red} \text{This solution misunderstood the range of } y \text{ for SOSC condition } y^{T}L\left ( x^{\ast },\mu ^{\ast } \right )y\geq 0 $

$ \color{red} y\in \tilde{T}\left( x^{* }\mu^{*} \right) \text{, where } \tilde{T}\left( x^{* },\mu^{*} \right)= \left \{ y:Dg_{i}\left( x^{*} \right)y=0, i\in \tilde{J}\left( x^{*},\mu^{*} \right) \right \} $

$ \color{red} \tilde{J}\left ( x^{\ast },\mu ^{\ast } \right ) \text{ has constrain for } \mu ^{\ast } \text{, } \tilde{J}\left ( x^{\ast },\mu ^{\ast } \right )= \left \{ i:g_{i}\left ( x^{\ast } \right ) = 0,\mu_{i}^{\ast }> 0\right \} $

$ \color{red} \tilde{J}\left ( x^{\ast },\mu ^{\ast } \right ) \subset J\left(x^{*}\right) $,    $ \color{red} T\left( x^{* } \right) \subset \tilde{T}\left( x^{* },\mu^{*} \right) $


$ \color{blue}\text{Solution 2:} $

$ L\left ( x_{1},\mu \right )= D^{2} l \left ( x _{1},\mu \right )= \begin{bmatrix} 2+2\mu_{1} & 0 \\ 0 & 2 \end{bmatrix} $

                   $ \text{for point } x^{*}=\begin{bmatrix} 0 \\ 0 \end{bmatrix} \text{, we get } \mu_{1}=-2 \text{ from KKT condition.} $

           $ \therefore L \left ( x^{*}, \mu ^{*}\right )=\begin{bmatrix} -2 & 0 \\ 0 & 2 \end{bmatrix} $

$ \tilde{T}\left( x^{* },\mu^{*} \right)= \left \{ y:Dg_{i}\left( x^{*} \right)y=0, i\in \tilde{J}\left( x^{*},\mu^{*} \right) \right \} $

             $ \tilde{J}\left ( x^{\ast },\mu ^{\ast } \right )= \left \{ i:g_{i}\left ( x^{\ast } \right ) = 0,\mu_{i}^{\ast }> 0\right \} $     $ \therefore i= 2 $

             $ \therefore \tilde{T}\left ( x^{\ast },\mu ^{\ast } \right )= \left \{ y:\left [ 1,1 \right ]y= 0 \right \}= \left \{ y:y_{1}= -y_{2} \right \} $

$ y^{T}L\left ( x^{\ast },\mu ^{\ast } \right )y=\begin{bmatrix} y_{1}& y_{2} \end{bmatrix}\begin{bmatrix} -2 & 0\\ 0 & 2 \end{bmatrix} \begin{bmatrix} y_{1}\\ y_{2} \end{bmatrix} \geqslant 0 $

                    $ -2y_{1}^{2}+2y_{2}^{2}\geqslant 0\cdots \left ( 1 \right ) $

                    for y1 = - y2,  (1) is always satisfied.

$ \therefore \text{For all } y\in \tilde{T}\left( x^{* },\mu^{*} \right) \text{, we have } y^{T}L\left ( x^{\ast },\mu ^{\ast } \right )y\geq 0 $

$ \therefore \text{point } x^{*} \text{satisfy the SOSC} $



$ \color{blue}\text{Related Problem: } $

                  minimize    − x2 + (x1 − 1)2 − 2

                  $ \text{subject to } x_{1}+x_{2}\leq2 $

$ \text{KKT condition: 1. } \mu\geq0 $

                                $ \text{2. } \begin{bmatrix} 2 \left(x_{1}-1 \right) & -1 \end{bmatrix} +\mu \begin{bmatrix} 1 & 1 \end{bmatrix} = \begin{bmatrix} 0 & 0 \end{bmatrix} $

                                        $ \text{3. } \mu \left( x_{1}+x_{2}-2 \right)=0 $

$ \text{From 2, } \mu=1 \text{, and } 2\left ( x_{1}-1 \right )=-1 $

$ \text{From 3, } \left( x_{1}+x_{2} \right) =2 $

$ \text{From above two equations, we obtain a candidate point for the minimizer } x^{*}=\begin{bmatrix} 1/2\\ 3/2 \end{bmatrix} $

Check for SOSC:

$ L \left ( x^{*}, \mu ^{*}\right )= F\left ( x^{*}\right )+ \mu ^{*} G\left ( x^{*}\right )=\begin{bmatrix} 2 & 0 \\ 0 & 0 \end{bmatrix} $

Because μ * > 0 ,whave

$ \tilde{T}\left( x^{* },\mu^{*} \right)= \left \{ y: \begin{bmatrix} 1 & 1 \end{bmatrix}y=0 \right \} = \left \{ y: =\begin{bmatrix} a & -a \end{bmatrix}:a\in\Re \right \} $

$ \text{Hence } y^{T}L\left ( x^{\ast },\mu ^{\ast } \right )y=\begin{bmatrix} a & -a \end{bmatrix}\begin{bmatrix} 2 & 0\\ 0 & 0 \end{bmatrix}\begin{bmatrix} a \\ -a \end{bmatrix}=2a^{2}>0 $

$ \therefore x^{*} \text{ satisfies the SOSC for strict local minimizer. } $



Automatic Control (AC)- Question 3, August 2011

Go to


Back to ECE Qualifying Exams (QE) page

Alumni Liaison

has a message for current ECE438 students.

Sean Hu, ECE PhD 2009